You are on page 1of 82

IASbabas Daily Quiz

November 1, 2016
Q.1) Agricultural Marketing and Farmer Friendly Reforms Index is released by
a)
b)
c)
d)

NITI Aayog
WTO
Food and Agriculture Organization of the United Nations
Indian Council of Agricultural Research

Q.1) Solution (a)


In News - http://www.livemint.com/Politics/0nw2jhsC3XMnByOC6VsyLJ/Maharashtratops-agri-marketing-farmer-friendly-reforms-ind.html

Q.2) Consider the following statements about National Committee on Trade Facilitation
(NCTF)
1. It has been constituted in line with the World Trade Organisation (WTOs) Trade
Facilitation Agreement (FTA), which India also has ratified
2. It will be responsible for identifying the nature of required legislative changes
needed for compliance with Trade Facilitation Agreement (TFA)
Select the correct statements:
a)
b)
c)
d)

Only 1
Only 2
Both 1 and 2
Neither 1 nor 2

Q.2) Solution (c)


The Union Government has constituted a National Committee on Trade Facilitation (NCTF)
to develop the pan-India road map for trade facilitation.
It has been constituted in line with the World Trade Organisation (WTOs) Trade Facilitation
Agreement (FTA), which India also has ratified.
It is an inter-ministerial body headed by cabinet secretary.
It will be responsible for identifying the nature of required legislative changes needed for
compliance with Trade Facilitation Agreement (TFA).

IASbabas Daily Quiz


November 1, 2016
Trade Facilitation It includes the simplification of procedures, reduction in time and cost,
augmentation of infrastructure and greater use of technology.
In News - http://www.thehindubusinessline.com/economy/india-needs-to-be-ready-toimplement-trade-facilitation-pact/article9282288.ece

Q.3) SIMBEX is maritime bilateral exercise between India and


a)
b)
c)
d)

Sri Lanka
Myanmar
Singapore
Australia and New Zealand

Q.3) Solution (c)


In News - http://www.business-standard.com/article/pti-stories/singapore-naval-shiprss-formidable-in-vizag-for-simbex-16-116103000372_1.html

Q.4) Which of the following is/are correctly matched?


City
1. Srinagar
2. Nashik
3. Guwahati

River
Jhelum
Ganga
Brahmaputra

Select the correct code:


a)
b)
c)
d)

1 and 2
1 and 3
2 and 3
All of the above

Q.4) Solution (b)


Nashik - Godavari
List of Indian cities on rivers https://en.wikipedia.org/wiki/List_of_Indian_cities_on_rivers

IASbabas Daily Quiz


November 1, 2016
Q.5) Andhra Pradesh and Telangana jointly topped Ease of Doing Business Reforms
Ranking 2015-16. It is jointly released by
a)
b)
c)
d)

DIPP and World Bank


Ministry of Corporate Affairs and WEF
WTO and WEF
NITI Aayog and DIPP

Q.5) Solution (a)


In News - http://www.livemint.com/Politics/SCiyKfQlC213Pu0Tg2d0qJ/Andhra-PradeshTelangana-joint-toppers-in-ease-of-doing-bus.html

IASbabas Daily Quiz


November 2, 2016
Q.1) With reference to H1B visa and L1 visa consider the following statements
1. L1 visa is a non-immigrant visa for short amount of time
2. H1B visa allows Indian employers to temporarily employ foreign workers in speciality
occupations
Select the correct statements:
a)
b)
c)
d)

Only 1
Only 2
Both 1 and 2
Neither 1 nor 2

Q.1) Solution (a)


The H-1B is a non-immigrant visa in the United States under the Immigration and Nationality
Act. It allows U.S. employers to temporarily employ foreign workers in specialty
occupations.
An L-1 visa is a visa document used to enter the United States for the purpose of work in L-1
status. It is a non-immigrant visa, and is valid for a relatively short amount of time, from
three months (for Iran nationals) to five years (India, Japan, and Germany), based on a
reciprocity schedule.With extensions, the maximum stay is seven years
In News - http://www.business-standard.com/article/economy-policy/bit-breakthroughunlikely-at-india-us-trade-meet-116101800488_1.html

Q.2) Which of the following is/are correctly matched?


1. Yamal Project :: Liquefied Natural Gas
2. Project Alloy :: Merged Reality
3. Project Tango :: 3D mapping technology
Select the correct code:
a)
b)
c)
d)

1 and 2
2 and 3
1 and 3
All of the above

IASbabas Daily Quiz


November 2, 2016
Q.2) Solution (d)
Project Tango is Googles take on three-dimensional mapping technology, but differs in that
it is intended to mimic the way humans navigate the world around them.
The mapping software works on an Android platform thats tricked out with image
processing and vision sensing technology. Project Tango uses depth sensors, visual clues and
motion tracking to essentially map the world around whoever is carrying the device.
Yamal is a liquefied natural gas project located deep in the Russian Arctic, a region that is
ice-bound for seven to nine months during the year and where the sun remains beneath the
horizon for three months at a time.

Q.3) Surya Kiran is a Joint Exercise between


a)
b)
c)
d)

India and Bhutan


India and Myanmar
India and Nepal
India and Maldives

Q.3) Solution (c)


In News - http://www.ndtv.com/india-news/indo-nepal-joint-military-exercise-kicks-off1587086

Q.4) Consider the following statements with respect to Permanent Court of Arbitration
(PCA)
1. It is the primary judicial branch of the United Nations
2. India is a party of the PCA according to the Hague Convention on 1899
3. Sessions of the PCA are held in private and are confidential
Which of the following statements is are correct?
a)
b)
c)
d)

Only 1
1 and 2
2 and 3
1 and 3

IASbabas Daily Quiz


November 2, 2016
Q.4) Solution (c)
The Permanent Court of Arbitration (PCA) is an intergovernmental organization located at
The Hague in the Netherlands. The PCA is not a court, but rather an organiser of arbitral
tribunals to resolve conflicts between member states, international organizations, or private
parties. It should not be confused with the International Court of Justice which is the
primary judicial branch of the United Nations, while the PCA is not a UN agency.
India is a party of the PCA according to the Hague Convention on 1899
List of parties to the Hague Conventions of 1899 and 1907 Click Here
Established by treaty at the First Hague Peace Conference in 1899, the Permanent Court of
Arbitration is the oldest global institution for the settlement of international disputes. The
Court offers a wide range of services for the resolution of international disputes which the
parties concerned have expressly agreed to submit for resolution under its auspices. Unlike
the International Court of Justice, the Permanent Court of Arbitration has no sitting judges:
the parties themselves select the arbitrators. Another difference is that sessions of the
Permanent Court of Arbitration are held in private and are confidential. The Court also
provides arbitration in disputes between international organisations and between states
and international organisations.
In News - http://indianexpress.com/article/world/australia-indonesia-mulling-jointpatrols-in-south-china-sea-3731673/

Q.5) Operation Sangaris is a military intervention of the French military in which of the
following countries?
a)
b)
c)
d)

Chad
Central African Republic
South Sudan
Congo

Q.5) Solution (b)


Operation Sangaris is a military intervention of the French military in the Central African
Republic, ongoing since December 2013.
In News - http://www.thehindu.com/todays-paper/tp-international/france-ends-carmilitary-mission/article9289369.ece

IASbabas Daily Quiz


November 3, 2016
Q.1) Consider the following statements about Yamuna river
1. It is the longest and the second largest tributary river of the Ganga
2. It flows through Uttarakhand, Himachal Pradesh and Uttar Pradesh
Select the correct statements
a)
b)
c)
d)

Only 1
Only 2
Both 1 and 2
Neither 1 nor 2

Q.1) Solution (c)


It crosses several states, Uttarakhand, Himachal Pradesh Haryana and Uttar Pradesh,
passing by Uttarakhand and later Delhi, and meets its tributaries on the way, including Tons,
its largest tributary in Uttarakhand, Chambal, its longest tributary which has its own large
basin, followed by Sindh, the Betwa, and Ken.
It is the longest and the second largest tributary river of the Ganges (Ganga) in northern
India. Originates from the Yamunotri Glacier
In News - http://economictimes.indiatimes.com/news/politics-and-nation/delhigovernment-launches-yamuna-riverfront-project/articleshow/55190207.cms

Q.2) Which of the following is not correctly matched?


a)
b)
c)
d)

TATP Europe and America


RCEP ASEAN
FTTA APEC
NAFTA Eurasian Union

Q.2) Solution (d)


NAFTA is the North American Free Trade Agreement is an agreement signed by Canada,
Mexico, and the United States, creating a trilateral trade bloc in North America.
FTTA is Free Trade Area of Asia Pacific championed by APEC.

IASbabas Daily Quiz


November 3, 2016
In News - http://www.livemint.com/Politics/mOzWKi4gvcNzK7KkBTqJWI/India-to-talktough-at-RCEP-trade-meet.html

Q.3) Which of the following pairs is/correctly matched?


1. Garba : Gujarat
2. Mohiniyattam: Kerala
3. Yakshagana: Karnataka
Select the correct codes
a)
b)
c)
d)

Only 1
2 and 3
1 and 3
All of the above

Q.3) Solution (d)


All are correctly matched
In News - http://www.thehindu.com/todays-paper/tp-national/tp-karnataka/weeklongchittani-yakshagana-saptaha-to-commence-in-udupi-from-tomorrow/article9225988.ece

Q.4) 'Saur Sujala Yojana' is concerned with


a)
b)
c)
d)

Solar powered irrigation pumps


24/7 electrified villages
Solar farms in Chhattisgarh
None of the above

Q.4) Solution (a)


Prime Minister Narendra Modi on Tuesday launched the Saur Sujala Yojana in Chhattisgarh
that would provide solar powered irrigation pumps to farmers at a subsidized price
Solar powered irrigation pumps of 3HP and 5HP capacity worth Rs 3.5 lakh and Rs 4.5 lakh
respectively would be distributed to the farmers
Chhattisgarh would be the first state to implement the scheme

IASbabas Daily Quiz


November 3, 2016
In News - http://www.business-standard.com/article/current-affairs/pm-narendra-modilaunches-saur-sujala-yojana-in-chhattisgarh-116110100699_1.html

Q.5) Consider the following statements about Fast Neutron Reactor


1. It is a category of nuclear reactor in which the fission chain reaction is sustained by
fast neutrons
2. There is no need of a neutron moderator
Select the correct statements
a)
b)
c)
d)

Only 1
Only 2
Both 1 and 2
Neither 1 nor 2

Q.5) Solution (c)


A fast neutron reactor or simply a fast reactor is a category of nuclear reactor in which the
fission chain reaction is sustained by fast neutrons. Such a reactor needs no neutron
moderator, but must use fuel that is relatively rich in fissile material when compared to that
required for a thermal reactor.
In News - http://www.thehindu.com/business/Industry/russia-invites-india-to-join-fastreactor-research-project/article9288233.ece

IASbabas Daily Quiz


November 4, 2016
Q.1) Which of the following is/are not a BASIC countries?
1.
2.
3.
4.

Indonesia
Cambodia
Bhutan
South Africa

Select the correct code:


a)
b)
c)
d)

1, 2 and 3
3 and 4
Only 4
All of the above

Q.1) Solution (a)


The BASIC countries (also Basic countries or BASIC) are a bloc of four large newly
industrialized countries Brazil, South Africa, India and China formed by an agreement on
28 November 2009. The four committed to act jointly at the Copenhagen climate summit,
including a possible united walk-out if their common minimum position was not met by the
developed nations.

Q.2) Consider the following statements about Uniting for Consensus (UfC)
1. It was developed to oppose the expansion of permanent seats of UNSC
2. Pakistan is one of the founding members of UfC
Select the correct statements
a)
b)
c)
d)

Only 1
Only 2
Both 1 and 2
Neither 1 nor 2

Q.2) Solution (c)


Indias nuclear-armed rival Pakistan has been leading the opposition to its inclusion in the
UNSCs list of permanent members. Other countries, part of an interest group called the
Uniting for Consensus (UfC), also curiously called The Coffee Club, formed in 1995, are
opposed to India (and the G4s bid) for permanent seats. Italy, Pakistan, Mexico and Egypt

IASbabas Daily Quiz


November 4, 2016
were founder members of the UfC. The list also includes Argentina, South Korea, Spain,
Turkey and Indonesia.

Q.3) Consider the following statements with respect to Schengen Area


1. Iceland, Norway and Switzerland are members of the Schengen Area but not the
European Union
2. No territory outside European Continent is part of the Schengen Area as well as
European Union
3. Post Brexit United Kingdom is now out of the Schengen Area
4. Even though the Schengen Agreement was signed in Luxembourg, it is not a member
of the Schengen Area
Which of the following statements is/are incorrect?
a)
b)
c)
d)

1, 2 and 4
2, 3 and 4
2 and 3
None of the Above

Q.3) Solution (b)


Iceland, Norway, and Switzerland - are associated members of the Schengen Area but are
not members of the European Union
Azores (Portugal), Madeira (Portugal), and the Canary Islands (Spain) these territories are
special members of the European Union and part of the Schengen Area, even that they are
located outside the European continent
Special member state territories and the European Union Click Here
United Kingdom never joined the Schengen Area
The Agreement was signed by the five (5) following European countries: France, Germany,
Belgium, Luxemburg, and Netherlands, in Schengen, a small village in Southern Luxemburg
on the river Moselle. Luxembourg is a member of the Schengen Area
Schengen Area Countries List Click Here
Interactive Map Click Here

IASbabas Daily Quiz


November 4, 2016
Q.4) Sampriiti-7 is a joint military exercise between
a)
b)
c)
d)

India, Bhutan and Bangladesh


India and Bhutan
India and Bangladesh
Nepal, Bhutan and India

Q.4) Solution (c)


In News - http://www.thehindu.com/todays-paper/tp-national/india-bangladesh-jointexercise-from-tomorrow/article9302566.ece

Q.5) Consider the following attributes of Mangroves


1.
2.
3.
4.

It is a glycophypte
It has aerial roots
Buffer zone between land and sea
West Bengal has the highest mangrove cover in India

Select the correct code


a)
b)
c)
d)

1,2 and 3
1,2 and 4
2,3 and 4
1,3 and 4

Q.5) Solution (c)


It is a halophypte Mangroves are salt tolerant trees, also called halophytes.
It has Aerial roots Pneumatophore
Mangroves are buffers between the land and the sea. Coastlines throughout the world are
facing serious problems of coastal erosion and threat of rising sea levels due to global
warming has increased the threats by several folds. To control such assault of the sea on
land the nature has provided what is called as Mangroves, a tropical littoral ecosystem
which is more dynamic than the sea itself.
Rank States/UTs with Highest Mangrove Cover 2013

Total Mangrove Cover in km2

IASbabas Daily Quiz


November 4, 2016
1
2
3
4
5
6
7
8
9
10

West Bengal
Gujarat
Andaman And Nicobar Islands
Andhra Pradesh
Odisha
Maharashtra
Tamil Nadu
Goa
Kerala
Karnataka

2,097
1,103
604
352
213
186
39
22
6
3

IASbabas Daily Quiz


November 7, 2016
Q.1) Druzhba 2016 is a joint military exercise between
a)
b)
c)
d)

Russia and China


Pakistan and Russia
Pakistan and China
None of the above

Q.1) Solution (b)


In News - http://indiatoday.intoday.in/story/brics-bimstec-2016-india-russiapakistan/1/787471.html

Q.2) Which of the following tribes inhabits the recently declared district of riverine island
of Majuli
a)
b)
c)
d)

Chenchu
Koli
Mishing
Bhil

Q.2) Solution (c)


The dwellers of Majuli are mostly of the Mising or Mishing tribes from Arunachal Pradesh
who immigrated here centuries ago. Apart from them, the inhabitants are from the Deori
and Sonowal Kacharis tribes. Languages spoken are Mising, Assamese, and Deori.

Q.3) Recently the Home Ministry launched the 'e-Court' system. Consider the following
statements
1. It is aimed at enabling speedy disposal of appeals and complaints by using digital
technology
2. It will offer an online search facility which allow users to search for dak, file, hearing
notice and decision
Select the correct statements
a) Only 1
b) Only 2

IASbabas Daily Quiz


November 7, 2016
c) Both 1 and 2
d) Neither 1 nor 2

Q.3) Solution (c)


The e-Court system to be launched during the inaugural function is aimed at enabling
speedy disposal of Appeals/Complaints by using digital technology.
The system will allow online filing of appeals/complaints and case disposals using latest
technology.
The system will generate a unique number which can be used by the applicant for tracking
the status of a case filed with the Commission.
As soon as a case is filed online or offline, the system will generate SMS/email alert to the
applicant.
Alerts will also be generated to inform the applicant at the stage of Registration, issue of
Facilitation Memo, scheduling of the case hearing and also at the stage of decision
uploading.
The system will offer an online search facility which allows a user to search for Dak, File,
Hearing Notice and Decision etc.
In News - http://www.business-standard.com/article/pti-stories/citizens-to-filecomplaints-appeals-online-in-cic-116110400709_1.html

Q.4) Consider the following statements about Square Kilometre Array (SKA)
1. It is an international effort to build the largest and the most sensitive radio telescope
in the world.
2. South Africas Karoo region and Bardez in Goa are the co-hosting locations
3. The NCRA-TIFR in Pune is the nodal institute for overseeing SKA related activities in
India
Which of the following statements is/are correct?
a)
b)
c)
d)

1 and 2
2 and 3
1 and 3
All of the above

IASbabas Daily Quiz


November 7, 2016

Q.4) Solution (c)


Square Kilometre Array is a large multi radio telescope project aimed to be built in Australia
and South Africa.
Both South Africas Karoo region and Western Australias Murchison Shire were chosen as
co-hosting locations for many scientific and technical reasons, from the atmospherics above
the desert sites, through to the radio quietness, which comes from being some of the most
remote locations on Earth.
South Africas Karoo desert will host the core of the high and mid frequency dishes,
ultimately extending over the African continent. Australias Murchison Shire will host the
low frequency antennas.
This International project aims to build worlds largest radio telescope for exploring the
Universe.
The radio telescope covers over one million square meters distance and it bring together a
wealth of the worlds finest scientist to the project.
Countries involved in this project: Australia, Canada, China, Italy, Netherlands, New Zealand,
South Africa, Sweden and UK.
SKA will hold its annual science meeting in Goa this year.
The National Centre for Radio Astrophysics (NCRA) in Pune is the nodal institute for
overseeing SKA related activities in India.
In News - http://www.thehindu.com/sci-tech/science/astronomers-to-gather-in-goa-forscience-meet/article9310348.ece

Q.5) Consider the following statements about Magnetospheric Multiscale Mission (MMM)
1. It is a joint mission by NASA and ESA
2. It holds the Guinness World Record for highest altitude fix of a GPS signal
3. Magnetic reconnection occurs when magnetic fields around Earth connect and
disconnect, explosively releasing energy
Select the correct statements
a) 1 and 2
b) 2 and 3

IASbabas Daily Quiz


November 7, 2016
c) 1 and 3
d) All of the above

Q.5) Solution (b)


NASAs MMS has set the Guinness World Record for highest altitude fix of a Global
Positioning System (GPS) signal above the surface of the Earth.
The primary focus for the MMS science team will be one of the most important and least
understood of those processes: Magnetic Reconnection. Magnetic reconnection occurs
when magnetic fields around Earth connect and disconnect, explosively releasing energy.
Understanding the causes of magnetic reconnection is important for understanding
phenomena around the universe from auroras on Earth, to flares on the surface of the sun,
and even to areas surrounding black holes.
In News - http://indianexpress.com/article/technology/science/nasas-mms-creates-newguinness-world-record-3739784/

IASbabas Daily Quiz


November 8, 2016
Q.1) Demchok region is in which of the following states?
a)
b)
c)
d)

Uttarakhand
Himachal Pradesh
Jammu & Kashmir
None of the above

Q.1) Solution (c)


In News - http://www.thehindu.com/news/national/armys-demchok-mission-asuccess/article9312345.ece

Q.2) Global Business Optimism Index is released by


a)
b)
c)
d)

World Bank
World Economic Forum
World Trade Organisation
None of the above

Q.2) Solution (d)


According to the most recent Grant Thornton International Business Report, India improved
its ranking by one spot in a global index of business optimism, ranking second during the
third quarter (July-September 2016). Through the April-June period, India was placed third
on the list.
In News - http://economictimes.indiatimes.com/news/economy/indicators/with-gst-onits-way-india-rises-to-second-spot-on-global-biz-optimismindex/articleshow/55277143.cms?utm_source=contentofinterest&utm_medium=text&ut
m_campaign=cppst

Q.3) Grey hounds have been in news recently, they are


a)
b)
c)
d)

Suicide bombers
Coastal guards
Anti-naxal force
Police dogs used in detecting bombs

IASbabas Daily Quiz


November 8, 2016

Q.3) Solution (c)


Guerrilla war-fare- They are specially trained for deep forest pursuit and combat.

Q.4) India is a member of which of the following Multilateral Export Control Regime
(MECR)?
a)
b)
c)
d)

Wassenaar Arrangement
Nuclear Suppliers Group
Missile Technology Control Regime
None of the above

Q.4) Solution (c)


Established in April 1987, the voluntary Missile Technology Control Regime (MTCR) aims to
limit the spread of ballistic missiles and other unmanned delivery systems that could be
used for chemical, biological, and nuclear attacks. The regime urges its 35 members,1 which
include most of the world's key missile manufacturers, to restrict their exports of missiles
and related technologies capable of carrying a 500-kilogram payload at least 300
kilometres or delivering any type of weapon of mass destruction.
The Wassenaar Arrangement on Export Controls for Conventional Arms and Dual-Use
Goods and Technologies
The Nuclear Suppliers Group (NSG), for the control of nuclear related technology
The Australia Group (AG) for control of chemical and biological technology that could be
weaponized
The Missile Technology Control Regime for the control of rockets and other aerial vehicles
capable of delivering weapons of mass destruction

Q.5) Gurgaon is renamed as Gurugram now. According to the Indian Constitution which of
the following Article was invoked by the Parliament to make the necessary changes?
a) Article 2
b) Article 3
c) Both (a) and (b)

IASbabas Daily Quiz


November 8, 2016
d) Article 1

Q.5) Solution (b)


Article 2 empowers the Parliament to admit into the Union of India, or establish, new
states on such terms and conditions as it thinks fit. Thus, Article 2 grants two powers to the
Parliament: (a) the power to admit into the Union of India new states; and (b) the power to
establish new states. The first refers to the admission of states which are already in
existence while the second refers to the establishment of states which were not in existence
before. Notably, Article 2 relates to the admission or establishment of new states that are
not part of the Union of India.
Article 3, on the other hand, relates to the formation of or changes in the existing states of
the Union of India. In other words, Article 3 deals with the internal re-adjustment inter se of
the territories of the constituent states of the Union of India.
Article 3 authorises the Parliament to:
a) form a new state by separation of territory from any state or by uniting two or more
states or parts of states or by uniting any territory to a part of any state,
b) increase the area of any state,
c) diminish the area of any state,
d) alter the boundaries of any state, and
e) alter the name of any state.

IASBabas Daily Quiz


November 9, 2016
Q.1) Which of the following constitute the World Bank?
1.
2.
3.
4.

International Bank for Reconstruction and Development


International Finance Corporation
International Development Association
International Monetary Fund

Choose the correct answer from the codes given below:


a)
b)
c)
d)

1, 2 and 3
1 and 2
3 and 4
1, 2, 3 and 4

Q.1) Solution (a)


The World Bank is an international financial institution that provides loans to developing
countries for capital programs.
It comprises two institutions: the International Bank for Reconstruction and Development
(IBRD) and the International Development Association (IDA).
IFC is a member of the World Bank Group. It finances and provides advice for private sector
ventures and projects in developing countries in partnership with private investors.
Link: The IMF and the World Bank: How Do They Differ?

Q.2) Which one of the following statement is true?


a)
b)
c)
d)

GDP is always lower than GNP


GDP is always higher than GNP
GDP is equal to GNP when depreciation is zero
None of the above

Q.2) Solution (d)


Gross Domestic Product (GDP) and Gross National Product (GNP) both try to measure the
market value of all goods and services produced for final sale in an economy. The difference
is how each term interprets what constitutes the economy. GDP refers to and measures the
domestic levels of production, whereas GNP measures the levels of production of any

IASBabas Daily Quiz


November 9, 2016
person or corporation of a country. For example, the American GNP measures the
production levels of any American or American-owned entity, regardless of where the actual
production process is taking place, and defines the economy in terms of the citizens. GNP is
less commonly referred to than GDP, but is best described as the measure of national
output.
Depending on circumstances, GNP can be either higher or lower than GDP. This depends on
the ratio of domestic to foreign manufacturers in a given country. For example, China's GDP
is $300 billion greater than its GNP, due to the large number of foreign companies
manufacturing in the country, whereas the GNP of the U.S. is $250 billion greater than its
GDP, because of the mass amounts of production that take place outside of the country's
borders.
Though both calculations attempt to measure the same thing, generally speaking, GDP is the
more commonly utilized method of measuring a country's economic success in the world,
but GNP can be useful as well. It is important to reference both when trying to get an
accurate description of a given country's economic worth.

Q.3) A multinational business:


a)
b)
c)
d)

Sells products abroad


Produces in more than one country
Imports from abroad
Sells only domestically

Q.3) Solution (b)


A multinational corporation (MNC) has facilities and other assets in at least one country
other than its home country. Such companies have offices and/or factories in different
countries and usually have a centralized head office where they coordinate global
management.

Q.4) Deficit Financing leads to


a)
b)
c)
d)

Recession
Boom
Inflation
Deflation

IASBabas Daily Quiz


November 9, 2016

Q.4) Solution (c)


Deficit financing may lead to inflation. Due to deficit financing money supply increases & the
purchasing power of the people also increase which increases the aggregate demand and
the prices also increases.

Q.5) Agricultural income tax is assigned to the State Governments by


a)
b)
c)
d)

the Finance Commission


the National Development Council
the Inter State Council
the Constitution of India

Q.5) Solution (d)


Entry 46 in the state list of seventh schedule

IASbabas Daily Quiz


November 10, 2016
Q.1) Consider the following statements about Liquefied Natural Gas (LNG)
1. It is colourless
2. It is odourless
3. Natural Gas can be liquefied without cooling it
Select the correct code:
a)
b)
c)
d)

Only 1
1 and 2
1 and 3
2 and 3

Q.1) Solution (b)


Natural gas is a mixture of hydrocarbons which, when liquefied, form a clear colourless and
odourless liquid; this LNG is usually transported and stored at a temperature very close to
its boiling point at atmospheric pressure (approximately 160 degree C).
LNG is very cold liquid form of natural gas - the fuel that's burned in gas stoves, home
heaters, and electric power plants. When it warms back up, LNG becomes natural gas again.
You can't liquefy natural gas without cooling it.
LNG does not burn itself .LNG needs to be in vapor form and mixed with air to burn .Is
combustible in the range of 5% to 15% volume concentrations in air .Combustible mixtures
in confined space will burn explosively .
LNG is a cryogenic substance and physical contact or spillage constitute a personnel and
equipment hazard. Natural gas presents an asphyxiation hazard. Its main component is
methane. It gasifies violently when directly introduced into a cargo tank at ambient
temperature, rapidly increases the internal pressure of the cargo tank and makes the
atmosphere into a flammable condition.

Q.2) Which of the following has the highest elasticity?


a)
b)
c)
d)

Steel
Copper
Rubber
Aluminium

IASbabas Daily Quiz


November 10, 2016
Q.2) Solution (a)

The definition of elastic in physics is unfortunately inverse of common sense elastic. The
more difficult it is to stretch, the more elastic a material is called to be because elasticity
is defined by the ratio stress to strain and not vice versa.
Elasticity is measured as ratio of stress to strain also known as Youngs Modulus. For a
given stress (stretching force per unit area) strain is much smaller in steel than in rubber
and hence the answer.
If the same force is applied to the wire of steel and rubber thread, which are of equal
length and cross-section area, we will find that the extension in the rubber thread is
much greater than extension in steel wire. Therefore, for a given stress, the strain
produced in steel is much smaller than that produced in the rubber. This implies that
Youngs modulus for steel is greater than that for rubber. Therefore, steel is more elastic
than rubber.
And also physics defines elasticity as "resistance to change". The greater the resistance
to change, the greater is the elasticity of the material and the faster it comes back to its
original shape or configuration when the deforming force is removed. By this definition,
steel is more elastic than rubber because steel comes back to its original shape faster
than rubber when the deforming forces are removed.

Q.3) Azerbaijan shares its border with which of the following countries?
1.
2.
3.
4.

Georgia
Iran
Armenia
Russia

Select the correct code:


a)
b)
c)
d)

1 and 3
1, 2 and 4
1, 3 and 4
All of the above

Q.3) Solution (d)

IASbabas Daily Quiz


November 10, 2016

Q.4) Consider the following statements with respect to Preston Curve


1. It is a relationship between development and health
2. Life expectancy is plotted against GDP per capita
Select the correct code
a)
b)
c)
d)

Only 1
Only 2
Both 1 and 2
Neither 1 nor 2

Q.4) Solution (c)

Its a relationship between development and health


Health of nations (life expectancy) plotted against wealth of nations (GDP per capita)
It says
Up to a point modest increase in GDP per capita corresponds to sharp
increase in life expectancy
After certain point, even large increases in public health expenditure leads to
modest increase in life expectancy

IASbabas Daily Quiz


November 10, 2016

Angus Deaton says

India is at/near the bend on the Preston Curve


Reasons
Infant mortality & maternal mortality still prevalent
Number of non-communicable diseases are increasing

Q.5) Networked Readiness Index is released by


a)
b)
c)
d)

Internet Governance Forum (IGF)


World Economic Forum (WEF)
World Wide Web Foundation
Working Group on Internet Governance (WGIG)

Q.5) Solution (b)


India was placed at 91st position in the recently released 2016 Networked Readiness Index
(NRI) by the Geneva-based World Economic Forum (WEF). The index was released as the
part of the WEFs Global Information Technology Report. It measures countries success in
creating the necessary conditions for a transition to a digitized economy and society.

IASbabas Daily Quiz


November 11, 2016
Q.1) Consider the following statements about Pradhan Mantri Yuva Yojana
1. It will focus on entrepreneurship training and education
2. It will be under Ministry of Skill Development and Entrepreneurship
Select the incorrect statements:
a)
b)
c)
d)

Only 1
Only 2
Both 1 and 2
Neither 1 nor 2

Q.1) Solution (c)


In News - http://pib.nic.in/newsite/PrintRelease.aspx?relid=153429

Q.2) Which of the following is not correctly matched?


Mission :: Study
1. RAVAN :: Clouds
2. MiRATA :: Temperature
3. HARP :: Earths Energy Budget
Select the correct code
a)
b)
c)
d)

Only 2
1 and 2
1 and 3
2 and 3

Q.2) Solution (a)


RAVAN, the Radiometer Assessment using Vertically Aligned Nanotubes, is a CubeSat that
will demonstrate new technology for detecting slight changes in Earths energy budget at
the top of the atmosphere. It will record essential measurements for understanding
greenhouse gas effects on climate.

IASbabas Daily Quiz


November 11, 2016
HARP, the Hyper-Angular Rainbow Polarimeter, will measure airborne particles and the
distribution of cloud droplet sizes with a new method that looks at a target from multiple
perspectives.
MiRaTA the Microwave Radiometer Technology Acceleration mission It has miniature
sensors that will collect data on temperature, water vapour and cloud ice that can be used
in weather forecasting and storm tracking.
In News - http://www.livemint.com/Science/Ig5WIUrneTwjMnN35u9weM/Nasa-tolaunch-6-small-lowcost-satellites-for-earth-monito.html

Q.3) One of the methods of Money Laundering is Round Tripping. Which of the following
statements regarding Round Tripping are correct?
1. Round Tripping refers to a practice of selling an unused asset to another company
while agreeing to buy back the same asset for about the same price.
2. Round-trip trading artificially inflates volume and revenues, and adds to large profits.
Select the code from the following:
a)
b)
c)
d)

1 only
2 only
Both 1 and 2
Neither 1 nor2

Q.3) Solution (a)


Round-trip trading is an action that attempts to inflate transaction volumes through the
continuous and frequent purchase and sale of a particular security, commodity or asset.
Round-trip trading can be used to refer to the practice of a business selling an unused asset
to another company while agreeing to buy back the same asset for about the same price.
This type of market manipulation has been seen in the energy and telecom business.
This is a market-manipulation practice used to misrepresent the number of transactions
occurring on any given day. Round-trip trading artificially inflates volume and revenues, but
in reality adds no profit. Enron was a company that engaged in round-trip trading, and, by
doing so, was able to increase revenues (and expenses) without changing its net income.
In News - http://www.livemint.com/Politics/Kr2JAc50rOWjhqldoijJ0M/Why-India-iscourting-Japan.html

IASbabas Daily Quiz


November 11, 2016

Q.4) Consider the following statements about Swing States with respect to US Elections
1. A swing state is a state in which no single candidate or party has overwhelming
support in securing the state's Electoral College votes
2. No candidate has an outright lead and the state could vote either way
3. Swing states vary by election years
Select the correct statements:
a)
b)
c)
d)

1 and 2
2 and 3
1 and 3
All of the above

Q.4) Solution (d)


How does the US election work and which swing states will determine the winner?
http://www.telegraph.co.uk/news/0/how-does-the-us-election-work-and-which-swingstates-will-determ/

Q.5) Which of the following is true about Psyche?


a)
b)
c)
d)

Saturns biggest moon


Largest metallic asteroid
A Kuiper belt object
None of the above

Q.5) Solution (b)


In News - http://timesofindia.indiatimes.com/home/science/water-detected-on-metallicasteroid-psyche/articleshow/55289301.cms

IASbabas Daily Quiz


November 14, 2016
Q.1) Consider the following statements
1. India is the largest recipient of Japanese official development assistance (ODA)
2. India is the first non-member of the non-proliferation treaty (NPT) to have signed
nuclear deal with Japan
Which of the following statements is/are incorrect?
a)
b)
c)
d)

Only 1
Only 2
Both 1 and 2
Neither 1 nor 2

Q.1) Solution (d)


Both the statements are correct.
Official development assistance (ODA) is a term coined by the Development Assistance
Committee (DAC) of the Organisation for Economic Co-operation and Development (OECD)
to measure aid. The DAC first used the term in 1969. It is widely used as an indicator of
international aid flow. It includes some loans.
Most ODA comes from the 28 members of the DAC, or about $135 billion in 2013. A further
$15.9 billion came from the European Commission and non-DAC countries gave an
additional $9.4 billion. Although development aid rose in 2013 to the highest level ever
recorded, a trend of a falling share of aid going to the neediest sub-Saharan African
countries continued.
In News - http://indianexpress.com/article/india/india-news-india/india-agrees-to-linknuclear-testing-to-termination-of-deal-with-japan-4372448/

Q.2) In order to capture day light and concentrate the same inside a dark room,
particularly in urban slum or rural areas which lack electricity supply, a low cost and
energy efficient Micro Solar Dome has been tested and developed. Consider the following
statements
1. It is a clear and green energy initiative of the Department of Science and
Technology
2. It is leak proof and works throughout the day and for a few hours continuously after
sunset

IASbabas Daily Quiz


November 14, 2016
Select the correct statements
a)
b)
c)
d)

Only 1
Only 2
Both 1 and 2
Neither 1 nor 2

Q.2) Solution (c)


Surya Jyoti - Photo-Voltaic (PV) Integrated Micro Solar Dome (MSD)
The Micro Solar Dome (MSD) is a day and night lighting single device unique in its features,
that has a transparent semi-spherical upper dome made of acrylic material which captures
the sunlight and the light passes through a sun-tube having a thin layer of highly reflective
coating on the inner wall of the passage. It also contains a lower dome made of acrylic.
There is a shutter in the bottom of the lower dome which can be closed, if light is not
required in the daytime. It is leak proof and works throughout the day and 4 hours
continuously after sunset. The entire development activities were taken up by an R&D
Organisation under the aegis of the Department of Science & Technology.
In News - http://dst.gov.in/pressrelease/surya-jyoti-photo-voltaic-pv-integrated-microsolar-dome-msd

Q.3) Consider the following statements about South Asian Satellite


1. All the members of SAARC are part of the project
2. It will be launched by GSLV MK III
Select the correct statements
a)
b)
c)
d)

Only 1
Only 2
Both 1 and 2
Neither 1 nor 2

Q.3) Solution (b)


It was previously named as SAARC Satellite. After Pakistan has opted out of this project, it
is renamed as South Asian Satellite

IASbabas Daily Quiz


November 14, 2016
In News - http://www.business-standard.com/article/current-affairs/south-asiansatellite-to-be-launched-in-march-2017-says-isro-116110800506_1.html

Q.4) Consider the following statements about Hunar Haat


1. It is aimed at promoting and supporting artisans from Minority communities and
providing them domestic as well as international market for display and sell their
products
2. It is being organized at India International Trade Fair
3. It is implemented by National Minorities Development & Finance Corporation
(NMDFC) under scheme USTTAD (Upgrading the Skills & Training in Traditional
Arts/Crafts for Development) of Ministry of Minority Affairs
Select the correct statements
a)
b)
c)
d)

1 and 2
2 and 3
1 and 3
All of the above

Q.4) Solution (d)


Hunar Haat

Hunar Haat (Skill Haat) exhibition is aimed at promoting and supporting artisans
from Minority communities and providing them domestic as well as international
market for display and sell their products.
The Hunar Haat is being organized at India International Trade Fair.
It is implemented by National Minorities Development & Finance Corporation
(NMDFC) under scheme USTTAD (Upgrading the Skills & Training in Traditional
Arts/Crafts for Development) of Ministry of Minority Affairs.

In News - http://pib.nic.in/newsite/PrintRelease.aspx?relid=153533

Q.5) Consider the following statements about Supermoon


1. It occurs when a full moon coincides with the moon being the closest it gets to the
Earth on its orbit
2. The main cause of this event is that the moons orbit around the Earth is elliptical

IASbabas Daily Quiz


November 14, 2016
Select the correct statements
a)
b)
c)
d)

Only 1
Only 2
Both 1 and 2
Neither 1 nor 2

Q.5) Solution (c)

In News - http://indianexpress.com/article/opinion/columns/supermoon-a-spectacle-inthe-sky-monday-night-4373712/

IASbabas Daily Quiz


November 15, 2016
Q.1) Consider the following statements about India International Trade Fair 2016
1. The Partner Country is the South Korea and the Focus Country is Japan
2. The Partner States are Rajasthan and Gujarat and the Focus State is Haryana
3. The theme of this year International trade fair is Digital India
Which of the following statements is/are correct?
a)
b)
c)
d)

1 and 2
Only 3
1 and 3
2 and 3

Q.1) Solution (b)


India International Trade Fair 2016

It is the annual flagship event of India Trade Promotion Organisation (ITPO).


The Partner Country is the South Korea and the Focus Country is Belarus.
The Partner States are Madhya Pradesh and Jharkhand and the Focus State is
Haryana.
The theme of this year International trade fair is Digital India.

In News - http://www.business-standard.com/article/government-pressrelease/incredible-india-india-international-trade-fair-2016-116111401415_1.html

Q.2) Which of the following statements concerning Solanum nigrum is/are correct?
1. A compound derived from the leaves of Solanum nigrum Uttroside B can treat
liver cancer
2. The compound has foaming characteristics
3. It is also called as Bittersweet Nightshade
Select the correct code:
a)
b)
c)
d)

1 and 2
1 and 3
2 and 3
All of the above

IASbabas Daily Quiz


November 15, 2016
Q.2) Solution (a)
Solanum nigrum

A compound derived from the leaves of Solanum nigrum, has remarkable efficacy in
treating liver cancer.
The compound saponin named Uttroside B, has shown that it is 10 times more
effective than Sorafenib, the only drug currently available for the treatment of
hepatocellular carcinoma (common type of liver cancer).
Saponins are glucosides with foaming characteristics
Solanum nigrum belongs to the family Solanaceae. It is also called as Black
Nightshade.
It is widely used in traditional medicine for various ailments such as inflammation,
jaundice, bronchitis, asthma, leprosy, and skin disorders. It is a rich source of
anticancer molecules.

In News - http://www.thehindu.com/news/cities/Thiruvananthapuram/herbalcompound-promises-cure-for-liver-cancer/article9342397.ece

Q.3) Naseem Al Bahr is a joint naval exercise between


a)
b)
c)
d)

Saudi Arabia and Pakistan


Turkey and Iran
India and Oman
Oman, Jordan and Turkey

Q.3) Solution (c)


Naseem Al Bahr is the military exercise between India and Oman.

Q.4) Consider the following statements about International Centre for Agricultural
Research in the Dry Areas (ICARDA)
1. ICARDA has been temporarily headquartered in Beirut, Lebanon
2. ICARDA, hosts the largest collection of seeds in Morocco
Select the correct statements
a) Only 1

IASbabas Daily Quiz


November 15, 2016
b) Only 2
c) Both 1 and 2
d) Neither 1 nor 2

Q.4) Solution (c)


The International Centre for Agriculture Research in the Dry Areas (ICARDA), a member of
the CGIAR, supported by the CGIAR Fund, is a non-profit agricultural research for
development institute that aims to improve the livelihoods of the resource-poor across the
worlds dry areas.
ICARDA has been temporarily headquartered in Beirut, Lebanon, since leaving Aleppo, Syria,
in 2012. Support is provided by research centres and offices in Jordan, Morocco, Ethiopia,
Egypt, Tunisia, Turkey, Afghanistan, India, Iran, Oman, Pakistan, Sudan, Uzbekistan, the
United Arab Emirates, and Yemen.
The International Centre for Agricultural Research in the Dry Areas, or ICARDA, hosts the
largest collection of seeds in Morocco.
The crucial role of seed banks in protecting biodiversity is receiving increasing attention
because of climate change, which threatens to wipe out crops as dry areas of the world get
even hotter and drier.
The Rabat centre holds tens of thousands of seeds spanning from wheat and barley to lentils
and chickpeas inside a vault in near-freezing temperatures.
The seed bank not only preserves these essential staples but develops them to become
more resistant to disease and a warming climate.
The site in Rabat has become primary centre of storage and research after its previous hub
in Aleppo, Syria, was seized by an Islamist rebel group in September 2015.

Q.5) Consider the following statements about The Consumer Protection Bill 2015
1. The Bill replaces the Consumer Protection Act, 1986
2. The Bill enforces consumer rights, and provides a mechanism for redressal of
complaints regarding defect in goods and deficiency in services
Select the correct statements
a) Only 1
b) Only 2

IASbabas Daily Quiz


November 15, 2016
c) Both 1 and 2
d) Neither 1 nor 2

Q.5) Solution (c)


The Bill replaces the Consumer Protection Act, 1986. The Bill enforces consumer rights, and
provides a mechanism for redressal of complaints regarding defect in goods and deficiency
in services.
Consumer Dispute Redressal Commissions will be set up at the district, state and national
levels for adjudicating consumer complaints.
The Bill establishes a Consumer Protection Authority to investigate into consumer
complaints, issue safety notices for goods and services, and pass orders for recall of goods
and against misleading advertisements.
If a consumer suffers an injury from a defect in a good, he may file a claim of product
liability against the manufacturer. The consumer must establish seven conditions in order
to prove such a claim.
The Bill classifies six contract terms as unfair. These cover terms such as (i) payment of
excessive security deposits; (ii) disproportionate penalty for a breach ; (iii) unilateral
termination without cause; (iv) one which puts the consumer at a disadvantage.

IASbabas Daily Quiz


November 16, 2016
Q.1) Consider the following statements about Indian Ocean Dipole (IOD)
1. It is an atmosphere-ocean coupled phenomenon in the tropical Indian Ocean
characterized by a difference in sea-surface temperatures (SST)
2. Positive IOD is characterized by warmer than normal SSTs in the eastern equatorial
Indian Ocean and cooler than normal SSTs in the western tropical Indian Ocean
3. Negative IOD is characterized by cooler than normal sea-surface temperatures in the
eastern equatorial Indian Ocean and warmer than normal sea-surface temperatures
in the western tropical Indian Ocean
4. Negative IOD has a more direct and immediate impact on the North-East monsoon
Which of the following statements is/are correct?
a)
b)
c)
d)

Only 1
1, 2 and 4
1 and 4
All of the above

Q.1) Solution (c)


IOD It is an atmosphere-ocean coupled phenomenon in the tropical Indian Ocean (like the
El Nino is in the tropical Pacific), characterized by a difference in sea-surface temperatures
(SST).
Positive IOD It is associated with cooler than normal sea-surface temperatures in the
eastern equatorial Indian Ocean and warmer than normal sea-surface temperatures in the
western tropical Indian Ocean.
Negative IOD It is characterized by warmer than normal SSTs in the eastern equatorial
Indian Ocean and cooler than normal SSTs in the western tropical Indian Ocean.
According to India Met Department (IMD), the North-East monsoon is likely to extend its
deficient run for next three days thanks to misdirected weather systems in the Bay of
Bengal/Indian Ocean.
This is in turn attributable to the persisting negative phase of the Indian Ocean Dipole (IOD),
which causes the Indian Ocean just south to Bay of Bengal to warm up abnormally.
This has direct implications for the North-East monsoon, since the warmth builds up lower
pressure over the Indian Ocean where most of the moisture gets directed.
In News - http://www.thehindubusinessline.com/economy/agri-business/is-thenortheast-monsoon-headed-for-a-washout/article9345836.ece

IASbabas Daily Quiz


November 16, 2016
Q.2) Consider the following statements about Hypersaline lakes
1.
2.
3.
4.

The water of hypersaline lakes has minimum buoyancy


Dead Sea is the largest hyper saline lake in the world
Araruama Lagoon in Brazil is the deepest hyper saline lake in the world
The salinity of hyper saline lakes is always less than that of ocean water

Which of the following statements is are incorrect?


a)
b)
c)
d)

1, 2 and 3
1, 3 and 4
1 and 4
All of the above

Q.2) Solution (d)


A hypersaline lake is a landlocked body of water that contains significant concentrations of
sodium chloride or other salts, with saline levels surpassing that of ocean water (3.5%, i.e.
35 grams per litre).
The most saline water body in the world is the Don Juan Pond, located in the McMurdo Dry
Valleys in Antarctica. Its volume is some 3,000 cubic meters, but is constantly changing. The
Don Juan Pond has a salinity level of over 44%, (i.e. 12 times saltier than ocean water). Its
high salinity prevents the Don Juan from freezing even when temperatures are below 50 C
(58 F). There are larger hypersaline water bodies, lakes in the McMurdo Dry Valleys such
as Lake Vanda with salinity of over 35% (i.e. 10 times saltier than ocean water). They are
covered with ice in the winter.
The water of hypersaline lakes has great buoyancy due to a high salt content.
The Dead Sea, dividing Israel and the Palestinian West Bank from Jordan, is the world's
deepest hypersaline lake and the Araruama Lagoon is the world's largest hypersaline lake.
In News - http://timesofindia.indiatimes.com/home/environment/flora-fauna/Climatehumans-caused-decline-of-once-2nd-largest-salt-lake/articleshow/55418684.cms

Q.3) Green GDP means


1. Conventional GDP figures adjusted for the environmental costs of economic
activities
2. Monetary value of the Forests

IASbabas Daily Quiz


November 16, 2016
3. Growth of Green Investments in economy
4. Accounts showing the monetized loss of biodiversity, costs caused by climate change
Select the correct code:
a)
b)
c)
d)

1 and 3
1 and 4
1, 3 and 4
All of the above

Q.3) Solution (b)


Green Gross Domestic Product is the index of the Economic growth of a particular country
which enshrines the environment consequences of the economic growth.
Green GDP does not mean the monetary value of the Forests etc.
Green GDP does not mean the growth of Green Investments.
Green GDP means that it accounts the monetized loss of biodiversity, costs caused by
climate change.
Green GDP is conventional gross domestic product figures adjusted for the environmental
costs of economic activities. Its a measure of how a country is prepared for sustainable
economic development.
This means that GDP may have some indicators such as Waste per capita or CO2 emissions
growth/ decline.
In News - http://www.thehindubusinessline.com/economy/green-gdp-plans-gatheringdust-in-government-corridors/article9345806.ece?homepage=true

Q.4) Consider the following statements with respect to Golden Mahseer Fish.
1.
2.
3.
4.

It is the longest-living freshwater fish


It is native to mountain and sub-mountain regions
It inhabit only rivers
It is an omnivore

Select the correct code


a) 1, 2 and 3

IASbabas Daily Quiz


November 16, 2016
b) 1, 2 and 4
c) 1 and 2
d) All of the above

Q.4) Solution (b)


Golden Mahseer Fish is the longest-living freshwater fish and is native to mountain and submountain regions.
It inhabit both rivers and lakes and is an omnivore
In News - http://timesofindia.indiatimes.com/city/dehradun/International-angling-meetkicks-off-in-Pancheswar/articleshow/55037119.cms
Read More
http://timesofindia.indiatimes.com/home/environment/flora-fauna/Himachal-Pradeshpropagating-mahseer-fish-for-conservation/articleshow/52348942.cms
https://en.wikipedia.org/wiki/Mahseer

Q.5) Consider the following statements:


1. Mixing fly ash in soil can help farmers increase production of crops and vegetables
2. Fly ash can be mixed with cement thus reducing the cost of construction.
3. Fly ash can cover the surface of plant leaves and thus helps in increasing nutrient
capacity.
4. Fly ash bricks are light weight and offer high strength and durability.
5. Use of fly-ash instead of lime in agriculture can reduce net CO2 emission and also
reduce global warming.
Which of the above statements are correct w.r.t Fly Ash?
a)
b)
c)
d)

1, 2 , 3 and 4
2, 3, 4 and 5
1, 2, 4 and 5
All of the above

Q.5) Solution (c)

IASbabas Daily Quiz


November 16, 2016
Fly ash in atmosphere acts as a pollutant. It can cover the leaf surface by making a thin layer
which reduces photosynthesis and productivity of plants. But its use in soils shows different
result. Fly ash is a resourceful material and can be effectively utilized as soil modifier in large
quantity and micro fertilizer in converting wasteland (barren land, rocky nature, sandy and
water logged soil, highly alkali and acidic soil etc.) into agriculturally productive land. "Best
thing about fly ash is, that it retains water in ground and helps bacterial actions to take
place to cultivate good quality of crops or vegetables.
Refer to these
http://scialert.net/fulltext/?doi=ajar.2010.1.14
http://timesofindia.indiatimes.com/city/bhopal/Fertilizer-costly-fly-ash-can-workmagic/articleshow/33835877.cms
In News - http://www.dnaindia.com/india/report-following-delhi-s-pollution-disastermaharashtra-adopts-policy-to-curb-transport-of-fly-ash-2273918

IASbabas Daily Quiz


November 17, 2016
Q.1) Which of the following is are correctly matched?
1. Hurricane Matthew : Haiti
2. Typhoon Lionrock : North Korea
3. Cyclone Winston : Fiji
Select the correct code:
a)
b)
c)
d)

1 and 3
2 and 3
2 and 3
All of the above

Q.1) Solution (d)


All are correctly matched

Q.2) Consider the following statements about Madanjeet Singh Prize


1. It is a prize awarded every two years by UNESCO
2. It rewards significant activities in the scientific, artistic, and cultural or
communication fields aimed at the promotion of a spirit of tolerance and nonviolence
Select the correct statements
a)
b)
c)
d)

Only 1
Only 2
Both 1 and 2
Neither 1 nor 2

Q.2) Solution (c)


In 1995, to mark the United Nations Year for Tolerance and the 125th anniversary of the
birth of Mahatma Gandhi, UNESCO created a prize for the promotion of tolerance and nonviolence.
The UNESCO-Madanjeet Singh Prize for the Promotion of Tolerance and Non-Violence
rewards significant activities in the scientific, artistic, and cultural or communication fields
aimed at the promotion of a spirit of tolerance and non-violence.

IASbabas Daily Quiz


November 17, 2016
The prize is awarded every two years on the International Day for Tolerance, 16 November.
The Prize may be awarded to institutions, organizations or persons, who have contributed in
a particularly meritorious and effective manner to tolerance and non-violence.
The 2016 winner is the Federal Research and Methodological Center for Tolerance
Psychology and Education (Tolerance Center) of Russia.
In News - http://www.unesco.org/new/en/social-and-human-sciences/themes/fightagainstdiscrimination/sv4/news/unesco_madanjeet_singh_prize_to_be_awarded_to_the_tolera
nce/

Q.3) The Sunway TaihuLight is a _____________ supercomputer


a)
b)
c)
d)

South Korean
Japanese
Chinese
Taiwanese

Q.3) Solution (c)


In News - http://timesofindia.indiatimes.com/China-wins-supercomputer-crown-foreighth-time/articleshow/55451990.cms

Q.4) Consider the following statements about INS Sumitra


1. It is the first Saryu class patrol vessel of the Indian Navy
2. It was a part of Operation Rahaat
Select the correct statements
a)
b)
c)
d)

Only 1
Only 2
Both 1 and 2
Neither 1 nor 2

Q.4) Solution (b)

IASbabas Daily Quiz


November 17, 2016
INS Sumitra is the fourth and last Saryu class patrol vessel of the Indian Navy, designed and
constructed by Goa Shipyard Limited. It is the presidential yacht of India. It is designed to
undertake fleet support operations, coastal and offshore patrolling, ocean surveillance and
monitoring of sea lines of communications and offshore assets and escort duties.
The ship has been deployed for multiple operational tasks, the most notable being
Operation Rahat, which entailed the evacuation of personnel of various nationalities from
war-torn Yemen in 2015.
In News - http://www.business-standard.com/article/government-press-release/inssumitra-visits-auckland-new-zealand-for-international-naval-review-2016116111601675_1.html

Q.5) Indian Strategic Petroleum Reserve (ISPR) are located in


1. Mangalore
2. Chandikhole
3. Visakhapatnam
Select the correct code:
a)
b)
c)
d)

1 and 3
2 and 3
Only 1
All of the above

Q.5) Solution (a)


The Strategic Petroleum Reserves in India is located in Vishakhapatnam, Mangalore and
Padur.
In News - http://www.business-standard.com/article/news-cm/strategic-petroleumreserves-at-3-locations-created-116111601466_1.html

IASbabas Daily Quiz


November 18, 2016
Q.1) Spratly Islands is located in which of the following
a)
b)
c)
d)

Danube River
South China Sea
Gulf of Maine
Pacific Ocean

Q.1) Solution (b)


The Spratlys are one of the major archipelagos in the South China Sea which complicate
governance and economics in this part of Southeast Asia due to their location in strategic
shipping lanes. The islands have no indigenous inhabitants, but offer rich fishing grounds
and may contain significant oil and natural gas reserves and as such are important to the
claimants in their attempts to establish international boundaries. Some of the islands have
civilian settlements, but of the approximately 45 islands, cays, reefs and shoals that are
occupied, all contain structures that are occupied by military forces from Malaysia, Taiwan
(ROC), China (PRC), the Philippines and Vietnam. Additionally, Brunei has claimed an
exclusive economic zone in the south-eastern part of the Spratlys, which includes the Louisa
Reef.
In News - https://www.ft.com/content/0cd8fc6c-ac86-11e6-9cb3-bb8207902122

Q.2) Consider the following statements with respect to Software-as-a-Service (SaaS)


1. It is a way of delivering applications over the Internet as a service
2. It is a cloud based service
3. SaaS customers have no hardware or software to buy, install, maintain, or update
Which of the following statements is/are incorrect?
a)
b)
c)
d)

Only 2
1 and 2
1 and 3
None of the above

Q.2) Solution (d)


All the statements are correct

IASbabas Daily Quiz


November 18, 2016
Cloud computing applications, or apps, are the cloud-based services also known as Software
as a Service (SaaS)
A cloud service is any resource that is provided over the Internet. The most common cloud
service resources are Software as a Service (SaaS), Platform as a Service (PaaS) and
Infrastructure as a Service (IaaS)
In
News
http://www.thehindubusinessline.com/info-tech/nasscom-growthrevisionanalysis/article9354072.ece
Read More - https://www.salesforce.com/in/saas/

Q.3) Recently Kyasanur Forest Disease was in the news. Consider the following statements
w.r.t it
1. It is a viral disease also known as Rodent fever
2. It is endemic to South Asia
3. Kyasanur forests are located in Andhra Pradesh
Select the correct code
a)
b)
c)
d)

Only 1
Only 2
1 and 3
2 and 3

Q.3) Solution (b)


It is known as Monkey Fever. Kyasanur Forests are located in Karnataka.
In News - http://timesofindia.indiatimes.com/city/goa/Seasons-first-KFD-case-inValpoi/articleshow/55361296.cms

Q.4) Montague-Chemsford reforms or the Councils Act of 1919 was based on which of the
following:
a)
b)
c)
d)

Duke Memorandum
Nehru Report
Congress Working Committee report
Mueller report

IASbabas Daily Quiz


November 18, 2016
Q.4) Solution (a)
Duke Memorandum is associated with Sir William Duke, a member of the English Round
Table Group and he had formulated a scheme which eventually became the basis of Joint
Report of Montague and Chelmsford.
In News - http://www.thehindu.com/books/literary-review/nistula-hebbar-inconversation-with-shashi-tharoor-on-his-new-book-an-era-of-darkness-the-britishempire-in-india/article9334224.ece

Q.5) National Security Guard (NSG) deals with which of the following?
a)
b)
c)
d)

Combat Terrorism
UN Peace Keeping
Insurgency
Communal Problems

Q.5) Solution (a)


National Security Guard (NSG) deals with Combat Terrorism.
The other operations are dealt by Rapid Action Force (RAF) under CRPF.
In News - http://timesofindia.indiatimes.com/city/chandigarh/Sikh-group-calls-for-probein-British-help-to-India-in-Operation-Blue-Star/articleshow/55276377.cms

IASbabas Daily Quiz


November 21, 2016
Q.1) Consider the following statements about Commission for Agricultural Costs & Prices
(CACP)
1. It recommends MSPs only for pulses and commercial crops
2. Cabinet Committee on Economic Affairs (CCEA) takes a final decision on the MSPs
after the feedback from CACP
Select the correct statements
a)
b)
c)
d)

Only 1
Only 2
Both 1 and 2
Neither 1 nor 2

Q.1) Solution (b)

It is mandated to recommend minimum support prices (MSPs) to incentivise the


cultivators to adopt modern technology and overall grain production.
Thus, MSP for major agricultural products are fixed by the government, each year,
after taking the recommendations of the Commission.
As of now, CACP recommends MSPs of 23 commodities, which comprise 7 cereals, 5
pulses, 7 oilseeds and 4 commercial crops.
After receiving the feed-back, the Cabinet Committee on Economic Affairs (CCEA) of
the Union government takes a final decision on the MSPs.

In News - http://www.financialexpress.com/economy/demonetisation-prices-at-retailagricultural-markets-stable/447662/

Q.2) Mission Madhumeha is under the


a)
b)
c)
d)

Ministry of Health & Family Welfare


Ministry of Ayush
Ministry of Agriculture
Ministry of Environment & Forests

Q.2) Solution (b)


Mission Madhumeha

IASbabas Daily Quiz


November 21, 2016
The Ministry of AYUSH is also going to launch Mission Madhumeha through Ayurveda on
the occasion.
The Mission will be implemented throughout the country through National Treatment
Protocol for effective management of Diabetes through Ayurveda.
It provides for free Ayurvedic health care services for the patients of Diabetes by organizing
health check-up camps, public lectures for creating awareness in public regarding Diabetes
etc.
In News - http://www.business-standard.com/article/government-press-release/rapidprogress-in-field-of-ayurveda-for-prevention-control-of-diabetes-116111801267_1.html

Q.3) Which of the following statements about sharia compliant finance is/are correct?
1. It is a banking activity that is consistent with the principles of Sharia and it is practical
application through the development of Islamic economics
2. It is a finance system based on the principles of not charging interest for loans
Select the correct code:
a)
b)
c)
d)

Only 1
Only 2
Both 1 and 2
Neither 1 nor 2

Q.3) Solution (c)


Islamic finance is equity-based, asset-backed, ethical, sustainable, environmentally- and
socially-responsible finance. It promotes risk sharing, connects the financial sector with the
real economy, and emphasizes financial inclusion and social welfare.
The following key principles guide Islamic Finance:
1) Prohibition of interest on transactions (riba)
2) Financing must be linked to real assets (materiality)
3) Engagement in immoral or ethically problematic businesses not allowed (e.g., arms
manufacturing or alcohol production)
4) Returns must be linked to risks.

IASbabas Daily Quiz


November 21, 2016
In News http://economictimes.indiatimes.com/industry/banking/finance/banking/sharia-bankingrbi-proposes-islamic-window-in-banks/articleshow/55524538.cms

Q.4) Which of the following is known as Orchid state of India?


a)
b)
c)
d)

Mizoram
Nagaland
Meghalaya
Arunachal Pradesh

Q.4) Solution (d)


Arunachal's rich and colorful orchids find a place of pride. Out of about a thousand species
of orchids in India, over 600 are to be found in Arunachal alone. Hence this state can rightly
be called the "Orchid Paradise" or Orchid state of India.

Q.5) Consider the following statements about Vector-borne diseases


1. Vectors are living organisms that can transmit infectious diseases between humans
or from animals to humans.
2. Chikungunya, Malaria and Dengue are vector-borne
3. Mosquitoes are the best known disease vector
Select the correct statements
a)
b)
c)
d)

1 and 2
2 and 3
1 and 3
All of the above

Q.5) Solution (d)


Vectors are living organisms that can transmit infectious diseases between humans or from
animals to humans. Many of these vectors are bloodsucking insects, which ingest diseaseproducing microorganisms during a blood meal from an infected host (human or animal)
and later inject it into a new host during their subsequent blood meal.

IASbabas Daily Quiz


November 21, 2016
Mosquitoes are the best known disease vector. Others include ticks, flies, sandflies, fleas,
triatomine bugs and some freshwater aquatic snails.
Mosquitoes
Aedes

Chikungunya
Dengue fever
Rift Valley fever
Yellow fever
Zika

Anopheles

Malaria

Culex

Japanese encephalitis
Lymphatic filariasis
West Nile fever

In News - http://timesofindia.indiatimes.com/city/ahmedabad/400-cases-of-vectorborne-diseases-till-November-12/articleshow/55444926.cms

IASbabas Daily Quiz


November 22, 2016
Q.1) Consider the following statements about INS Chennai
1. It is indigenously designed guided missile destroyer in the Shivalik class
2. It is armed with supersonic surface-to-surface BrahMos missiles and Barak-8 long
range surface-to-air missiles
3. It was a part of Project 15A
Select the correct statements
a)
b)
c)
d)

1 and 2
2 and 3
1 and 3
All of the above

Q.1) Solution (b)


INS Chennai, the third indigenously designed guided missile destroyer in the Kolkata class
commissioned
The ship is armed with supersonic surface-to-surface BrahMos missiles and Barak-8 long
range surface-to-air missiles.
With an overall length of 164 metres and displacement of over 7,500 tonnes, INS Chennai
is one of the largest destroyers in the Indian Navys fleet.
Built at the Mazagon Dock Shipbuilders Ltd in Mumbai, the ships construction also marks
the end of the Project 15A to build Kolkata class guided missile destroyers.
In News - http://www.thehindu.com/news/national/Manohar-Parrikar-commissionsdestroyer-%E2%80%98INS-Chennai%E2%80%99/article16669047.ece

Q.2) Consider the following statements


1. There are no institutions under the administrative control of Ministry of Culture
named as Cultural Universities
2. Central Institute of Buddhist Studies (CIBS), Leh is declared as deemed to be
Universities under Ministry of Culture
Which of the following statements is are correct?
a) Only 1
b) Only 2

IASbabas Daily Quiz


November 22, 2016
c) Both 1 and 2
d) Neither 1 nor 2

Q.2) Solution (c)


There are four teaching institutions declared as deemed to be Universities under Ministry of
Culture, Government of India.
They are

Nava Nalanda Mahavihara (NNM), Nalanda


Central Institute of Buddhist Studies (CIBS), Leh
Central University of Tibetan Studies (CUTS), Varanasi
National Museum Institute, New Delhi

In News - http://pib.nic.in/newsite/mbErel.aspx?relid=153996

Q.3) Which of the following statements about New Energy Policy is/are correct?
1. It builds on the previous Integrated Energy Policy
2. It aligns the domestic coal prices with the international prices
3. It includes issues related to sharp decline of crude oil prices and change in solar
energy technology
Select the correct code:
a)
b)
c)
d)

1 and 2
2 and 3
1 and 3
All of the above

Q.3) Solution (c)


NEP builds on the achievements of the earlier omnibus energy policy the Integrated Energy
Policy (IEP), and sets the new agenda consistent with the redefined role of emerging
developments in the energy world.
Clearing a misconception about the aligning of domestic coal prices with the international
prices, Shri Goyal informed that the proposed policy does not call for any such alignment.

IASbabas Daily Quiz


November 22, 2016
The four key objectives of the new energy policy are access at affordable prices, improved
energy security and independence, greater sustainability and economic growth.
The new policy differs from the previous policy while including the issue related to sharp
decline of crude oil prices, change in solar energy technology, heightened concern of climate
change issues, ambitious target of Renewable energy and rural electrification agenda
adopted by the Government.
In News - http://pib.nic.in/newsite/PrintRelease.aspx?relid=153952

Q.4) Consider the following statements about European Organisation for Nuclear
Research (CERN)
1. India is an associate member of CERN
2. Only European countries are granted full membership
Select the correct statements
a)
b)
c)
d)

Only 1
Only 2
Both 1 and 2
Neither 1 nor 2

Q.4) Solution (a)


Israel is the only non-European country granted full membership.
In News - http://www.livemint.com/Science/xRZDQSCQ71qOUd8gxxOpbI/India-becomesassociate-member-of-CERN.html

Q.5) Consider the following statements about E-Kuber


1. It is the Core Banking Solution of Reserve Bank of India
2. It enables banks to offer a multitude of customer-centric services on a 247 basis
from a single location
3. The e-kuber system can be accessed either through INFINET or Internet
Select the correct statements
a) 1 and 2

IASbabas Daily Quiz


November 22, 2016
b) 2 and 3
c) 1 and 3
d) All of the above

Q.5) Solution (d)


e-Kuber is the Core Banking Solution of Reserve Bank of India. E-Kuber provides the
provision of a single current account for each bank across the country, with decentralised
access to this account from anywhere-anytime using portal based services in a safe manner.
Core Banking Solutions (CBS) can be defined as a solution that enables banks to offer a
multitude of customer-centric services on a 24x7 basis from a single location, supporting
retail as well as corporate banking activities, as well as all possible delivery channels existing
and proposed. The centralisation thus makes a one-stop shop for financial services a
reality. Using CBS, customers can access their accounts from any branch, anywhere,
irrespective of where they have physically opened their accounts.
The e-kuber system can be accessed either through INFINET or Internet. The INFINET is a
Closed User Group Network for the exclusive use of member banks and financial institutions
and is the communication backbone for the National Payments System, which caters mainly
to inter-bank applications like Real Time Gross Settlement (RTGS), Delivery Vs Payment,
Government Transactions, Automatic Clearing House, etc.
In News - http://www.thehindu.com/news/cities/puducherry/Govt.-to-issue-datedsecurities-worth-Rs.-125cr./article16662047.ece?utm_source=RSS_Feed&utm_medium=RSS&utm_campaign=RSS_
Syndication

IASbabas Daily Quiz


November 23, 2016
Q.1) Consider the following statements about Prithvi-II
1. It is the first missile to be developed by the DRDO under the Integrated Guided
Missile Development Programme
2. It is thrusted by liquid propulsion twin engines
3. It is a series of short range ballistic missiles launched to combat opponents in warlike situations
Which of the following statements is/are correct?
a)
b)
c)
d)

1 and 2
1 and 3
2 and 3
All of the above

Q.1) Solution (d)


Prithvi-II is the first missile to be developed by the DRDO under the Integrated Guided
Missile Development Programme
Prithvi II is a series of short range ballistic missiles launched to combat opponents in war-like
situations.
The first test fire of this missile was carried out in 1996 and its development stage was
completed in the year 2004.
It was inducted into Indian armed forces in 2003 by the Strategic Forces Command (SFC)
Prithvi-II missile has a strike range of 350 km
It is capable of carrying 500-1000 kg of warheads
Prithvi-II is thrusted by liquid propulsion twin engines. It is 9 metres tall.
It uses advanced inertial guidance system with manoeuvring trajectory to hit its target. Its
route was tracked by the DRDO radars, electro-optical tracking systems and telemetry
stations
Launch activities are carried out by the specially formed strategic force command (SFC) and
monitored by the scientists of Defence Research and Development Organisation (DRDO)
In a similar trial, India test fired Prithvi II as a part of a users trial in May.

IASbabas Daily Quiz


November 23, 2016
In News - http://www.thehindu.com/news/national/Prithvi-II-missiles-successfully-testfired/article16674321.ece

Q.2) Which of the following statements about Climate Change Performance Index (CCPI)
is/are correct?
1. It is released by United Nations Framework Convention on Climate Change (UNFCC)
and Intergovernmental Panel on Climate Change (IPCC)
2. It includes emissions from deforestation and forest degradation
Select the correct code:
a)
b)
c)
d)

Only 1
Only 2
Both 1 and 2
Neither 1 nor 2

Q.2) Solution (b)


The Climate Change Performance Index (CCPI) is an annual publication by Germanwatch and
Climate Action Network Europe
The Climate Change Performance Index is an instrument designed to enhance transparency
in international climate politics
Its aim is to put political and social pressure on those countries which have, up until now,
failed to take ambitious action on climate protection. It also aims to highlight those
countries with best-practice climate policies.
On the basis of standardised criteria, the index evaluates and compares the climate
protection performance of 58 countries that together are responsible for more than 90% of
global energy-related CO2 emissions
It includes emissions from deforestation and forest degradation
In News - http://www.business-standard.com/article/current-affairs/india-ranks-20th-onclimate-change-performance-index-116111701669_1.html

Q.3) Indias longest expressway connects

IASbabas Daily Quiz


November 23, 2016
a)
b)
c)
d)

Mumbai and Pune


Agra and Greater Noida
Lucknow and Agra
Ahmedabad and Vadodara

Q.3) Solution (c)


The Agra-Lucknow Expressway is six-lane green-field expressway having total length of 302km. It has a design speed of up to 120 kms per hour.
In News - http://www.financialexpress.com/india-news/agra-lucknow-expressway-indialongest-greenfield-highway-10-facts-features-highlights-akhilesh-yadav-upgovernment/451101/

Q.4) Consider the following statements w.r.t Kimberley Process Certification Scheme
(KPCS)
1. It is a joint initiative of the governments, industry and civil societies to prevent the
entry of conflict diamonds from the mainstream rough diamond market
2. India is founder member of the scheme
3. The current chair and vice chair of KPCS is UAE and India respectively
Select the correct code:
a)
b)
c)
d)

1 and 2
2 and 3
1 and 3
All of the above

Q.4) Solution (a)


The Chair rotates annually. The Vice Chair is selected at the annual plenary' meeting and
becomes Chair automatically the following year.
KPCS is a joint initiative of the governments, industry and civil societies to prevent the entry
of conflict diamonds from the mainstream rough diamond market.
It was established in 2003 through a United Nations General Assembly Resolution. India is
founder member of the scheme.

IASbabas Daily Quiz


November 23, 2016
In India, the scheme is administered through the Department of Commerce under the aegis
of the Union Commerce and Industry Ministry.
In News - http://pib.nic.in/newsite/PrintRelease.aspx?relid=153909

Q.5) Consider the following statements about Export Promotion Councils (EPC)
1. The EPCs are non-profit organisations registered under the Companies Act or the
Societies Registration Act
2. It performs only advisory functions
3. It may be provided financial assistance by the Central Government

IASbabas Daily Quiz


November 23, 2016
Select the correct statement
a)
b)
c)
d)

1 and 2
2 and 3
1 and 3
All of the above

Q.5) Solution (c)


The basic objective of Export Promotion Councils is to promote and develop the exports of
the country. Each Council is responsible for the promotion of a particular group of products,
projects and services.
The main role of the EPCs is to project India's image abroad as a reliable supplier of high
quality goods and services. In particular, the EPCs shall encourage and monitor the
observance of international standards and specifications by exporters. The EPCs shall keep
abreast of the trends and opportunities in international markets for goods and services and
assist their members in taking advantage of such opportunities in order to expand and
diversify exports.
The EPCs are non-profit organisations registered under the Companies Act or the Societies
Registration Act, as the case may be.
THE EPCs perform both advisory & executive functions.
Read More - http://pib.nic.in/archieve/eximpol/eximpol00-01/Chap-13.htm

IASbabas Daily Quiz


November 24, 2016
Q.1) Consider the following statements about Fame India Scheme
1. It comes under National Automotive Board, Department of Heavy Industry
2. It is aimed at incentivising all vehicle segments i.e. 2 Wheeler, 3 Wheeler Auto,
Passenger 4 Wheeler Vehicle, Light Commercial Vehicles and Buses
Which of the following statements is/are incorrect?
a)
b)
c)
d)

Only 1
Only 2
Both 1 and 2
Neither 1 nor 2

Q.1) Solution (d)


Government of India has notified FAME India Scheme [Faster Adoption and Manufacturing
of (Hybrid &) Electric Vehicles in India] with effect from April 2015.
It comes under National Automotive Board, Department of Heavy Industry.
Objective: to support hybrid/electric vehicles market development and Manufacturing ecosystem.
Four focus areas of FAME
1.
2.
3.
4.

Technology development
Demand Creation
Pilot Projects
Charging Infrastructure

The FAME Scheme is aimed at incentivising all vehicle segments i.e. 2 Wheeler, 3 Wheeler
Auto, Passenger 4 Wheeler Vehicle, Light Commercial Vehicles and Buses.
Scheme covers Hybrid & Electric technologies like Mild Hybrid, Strong Hybrid, Plug in Hybrid
& Battery Electric Vehicles.
Under this scheme, about 99000 hybrid/electric vehicles (xEVs) have been given direct
support by way of demand incentives since the launch
For details: http://www.fame-india.gov.in/
In News - http://pib.nic.in/newsite/mbErel.aspx?relid=154119

IASbabas Daily Quiz


November 24, 2016
Q.2) Consider the following statements about Pre-paid Payment Instruments (PPIs)
1. It facilitates purchase of goods and services, including funds transfer, against the
value stored on such instruments
2. Reserve Bank of India (RBI) is the regulatory authority for PPIs
Select the correct statements
a)
b)
c)
d)

Only 1
Only 2
Both 1 and 2
Neither 1 nor 2

Q.2) Solution (c)


Pre-paid Payment Instruments (PPIs) are defined in the RBI Guidelines issued under the
Payment and Settlements Systems Act, 2005 as payment instruments that facilitate
purchase of goods and services, including funds transfer, against the value stored on such
instruments. The value stored on such instruments represents the value paid for by the
holders by cash, by debit to a bank account, or by credit card. The pre-paid instruments can
be issued as smart cards, magnetic stripe cards, internet accounts, internet wallets, mobile
accounts, mobile wallets, paper vouchers and any such instrument which can be used to
access the pre-paid amount.
PPIs are one component of the Payment System. In fact, they constitute the last end of the
spectrum of payment instruments existing in India at the moment. The Payment and
Settlement Systems Act, 2007 (PSS Act, 2007) which came into force with effect from 12
August 2008, provides for the regulation and supervision of PPIs in India. Reserve Bank of
India (RBI) is the regulatory authority for this purpose.
In News - http://economictimes.indiatimes.com/news/economy/policy/rbi-doublesprepaid-payment-instruments-limit-to-rs-20000/articleshow/55567364.cms

Q.3) Consider the following statements about National Plant Protection Organization
(NPPO)
1. It is the nodal agency for inspecting the mills and granting certificates on plant health
for export purposes
2. A phytosanitary certificate for export is issued by NPPO
Select the correct statements

IASbabas Daily Quiz


November 24, 2016
a)
b)
c)
d)

Only 1
Only 2
Both 1 and 2
Neither 1 nor 2

Q.3) Solution (c)


Phytosanitary certification is used to attest that consignments meet phytosanitary
(regarding plants) import requirements and is undertaken by an NPPO (National Plant
Protection Organization).
To export to nations, it is mandatory that Indian rice exporters are registered with NPPO,
the Indian government agency for inspecting the mills and granting certificates on plant
health for export purposes.
In News - http://www.thehindu.com/business/Economy/China-agrees-to-import-ricefrom-17-mills-in-India/article16680955.ece?utm_source=RSS_Feed=RSS=RSS_Syndication

Q.4) What is Integrated Goods and Services Tax?


a)
b)
c)
d)

Tax imposed on imported goods and services


Tax imposed on interstate trade
Tax on international trade
Tax imposed on value additions to exports

Q.4) Solution (b)


IGST or integrated goods and services tax would mean the tax levied under IGST Act on the
supply of any goods and / or services in the course of inter-state trade or commerce.
In News - http://www.livemint.com/Politics/iV8o1uNhM2X4vtl2j8zFGK/Interstate-salesissue-trips-progress-on-GST.html

Q.5) Global Urban Ambient Air Pollution Database is released by


a) WHO
b) Global Atmosphere Watch (GAW)
c) Intergovernmental Panel on Climate Change (IPCC)

IASbabas Daily Quiz


November 24, 2016
d) None of the above

Q.5) Solution (a)


In News - http://www.thehindu.com/data/The-Indian-cities-with-the-poorest-airquality/article16688008.ece

IASbabas Daily Quiz


November 25, 2016
Q.1) Which of the following releases ICT Development Index (IDI)
a)
b)
c)
d)

International Telecommunications Union (ITU)


World Economic Forum
UN ICT Task Force
International Development Informatics Association (IDIA)

Q.1) Solution (a)


In News - http://tech.firstpost.com/news-analysis/over-forty-percent-of-the-populationin-india-does-not-own-even-a-basic-feature-phone-itu-report-349212.html

Q.2) Consider the following statements about Intertropical Convergence Zone (ITCZ)
1. The area encircling the earth near the equator where the northeast and southeast
trade winds come together is known as ITCZ
2. The zone experiences low precipitation and high humidity
Select the correct statements
a)
b)
c)
d)

Only 1
Only 2
Both 1 and 2
Neither 1 nor 2

Q.2) Solution (a)


The Intertropical Convergence Zone (ITCZ), known by sailors as the doldrums, is the area
encircling the earth near the equator where the northeast and southeast trade winds come
together.
This zone experience high precipitation and high humidity.

Q.3) Which of the following countries do not border Lake Victoria of Africa?
1.
2.
3.
4.

Tanzania
Uganda
Zaire
Rwanda

IASbabas Daily Quiz


November 25, 2016
Select the correct code:
a)
b)
c)
d)

1, 3 and 4
3 and 4
2 and 4
1 and 2

Q.3) Solution (b)

Q.4) Union Cabinet approved the new Merchant Shipping Bill, 2016 for introducing it in
the Parliament. Consider the following statements
1. It will repeal Merchant Shipping Act, 1958 as well as The Coasting Vessels Act, 1838
2. The Coasting Vessels Act, 1838 provides for registration of non-mechanically
propelled vessels to a limited jurisdiction of Saurashtra and Kutch
Select the correct statements
a)
b)
c)
d)

Only 1
Only 2
Both 1 and 2
Neither 1 nor 2

Q.4) Solution (c)

IASbabas Daily Quiz


November 25, 2016
The Bill is a revamped version of the Merchant Shipping Act, 1958. The Bill will repeal
Merchant Shipping Act, 1958 as well as The Coasting Vessels Act, 1838. The Merchant
Shipping Act, 1958 amended 17 times between 1966 and 2014 resulting in an increase in the
number of sections to more than 560 sections. These provisions have been meticulously
shortened to 280 sections in the new Bill.
The Coasting Vessels Act, 1838, an archaic legislation of the British era providing for
registration of non-mechanically propelled vessels to a limited jurisdiction of Saurashtra and
Kutch, is proposed to be repealed since provisions have been introduced in the Merchant
Shipping Bill 2016 for registration of all vessels for the whole of India.
In News - http://pib.nic.in/newsite/PrintRelease.aspx?relid=154201

Q.5) CFTRI is trying to bring about an omega revolution in India by developing protein
enriched and Omega-3 enriched chocobars, by adding chia and quinoa seeds to chocobars
help reduce fat and sugar. Consider the following statements w.r.t Omega-3 fatty acids
1. Omega-3 comes only from plant sources
2. Mammals are unable to synthesize omega-3 fatty acids
Select the correct statements
a)
b)
c)
d)

Only 1
Only 2
Both 1 and 2
Neither 1 nor 2

Q.5) Solution (b)


Omega-3 comes from both animal and plant sources. The primary animal sources are krill oil
and fish oil. The primary plant sources are flaxseed, chia and hemp.
Omega-3 fatty acids are important for normal metabolism. Mammals are unable to
synthesize omega-3 fatty acids, but can obtain the shorter-chain omega-3 fatty acid ALA (18
carbons and 3 double bonds) through diet and use it to form the more important long-chain
omega-3 fatty acids, EPA (20 carbons and 5 double bonds) and then from EPA, the most
crucial, DHA (22 carbons and 6 double bonds).
In News - http://www.thehindubusinessline.com/news/science/cftri-to-bring-about-anomega-revolution-in-india/article9382696.ece

IASbabas Daily Quiz


November 28, 2016
Q.1) Recently the Ministry of Agriculture launched e-pashuhaat portal. Consider the
following with respect to same.
1. The portal has been developed under National Mission on Bovine Productivity
2. It facilitates trade of live cows/buffaloes, frozen semen and embryos
3. The portal has real-time authentic certified information on availability of germplasm
Which of the following statements is/are correct?
a)
b)
c)
d)

1 and 2
1 and 3
2 and 3
1, 2 and 3

Q.1) Solution (d)


e-pashuhaat portal has been developed under National Mission on Bovine Productivity for
connecting breeders and farmers regarding availability of bovine germplasm. Through the
portal breeders/farmers can sell and purchase breeding stock, information on all forms of
germplasm including semen, embryos and live animals with all the agencies and stake
holders in the country has been uploaded on the portal. Through this portal, farmers will be
aware about the availability of quality disease free bovine germplasm with different
agencies in the country. The portal will lead to propagation of high genetic merit
germplasm.
The portal has real-time authentic certified information on availability of germplasm,
establishes links between 'farmer to farmer' and 'farmer to institutes' and minimises the
involvement of middlemen.
India has the largest bovine population in the world. There are 144 million cattles and 105
million buffaloes. About 79 per cent are of indigenous breeds while the rest 21 per cent are
exotic and crossbreed varieties.
In News
http://www.business-standard.com/article/pti-stories/8-62l-frozen-bovine-semen-soldin-hours-of-e-pashu-haat-launch-116112600333_1.html
http://pib.nic.in/newsite/PrintRelease.aspx?relid=154398

IASbabas Daily Quiz


November 28, 2016
Q.2) Consider the following statements about Lapis Lazuli Corridor
1. The corridor connects Afghanistan, Turkmenistan, Azerbaijan, Georgia and Turkey via
road and rail
2. It starts from Afghanistan and ends in Europe
3. It is derived from the historic export route that Afghanistans lapis lazuli and other
semiprecious stones were being exported through this route to Caucasus, Russian,
the Balkans, Europe and North of Africa over 2000 years back
Select the correct statements
a)
b)
c)
d)

1 and 2
2 and 3
1 and 3
All of the above

Q.2) Solution (d)


Lapis Lazuli Corridor is created toward transit and trade cooperation among Afghanistan,
Turkmenistan, Azerbaijan, Georgia and Turkey intended to reduce barriers facing transit
trade and develop a Custom Procedure Integration in the region.
The corridor connects Afghanistan, Turkmenistan, Azerbaijan, Georgia and Turkey via road
and rail that is most appropriate transit trade route in Central Asia, the Caucasus, the
Balkans and Central Europe and also connects effectively South Asia to European countries.
The Lapis Lazuli begins from Aqina in northern Faryab province and Turqundi in western
Herat province of Afghanistan and continues to Turkmenbashi of Turkmenistan and after
passing Caspian Sea, arrives Baku, the Azerbaijans capital and then it connects Baku to
Tbilisi, Georgias capital and also to the ports of Polti and Batumi of Georgia. And then get
cities of Kars and Istanbul of Turkey and finally ends Europe.
The Lapis Lazuli is derived from the historic export route that Afghanistans lapis lazuli and
other semiprecious stones were being exported through this route to Caucasus, Russian, the
Balkans, Europe and North of Africa over 2000 years back.
In News - http://indianexpress.com/article/world/world-news/pakistan-joins-ashgabatagreement-lapis-lazuli-corridor-4396500/

IASbabas Daily Quiz


November 28, 2016
Q.3) Joint defence exercises Indra-Neva, AviaIndra and Indra are between India and
a)
b)
c)
d)

Only Russia
US and Japan
Japan, Vietnam and Philippines
Russia US and Japan

Q.3) Solution (a)

Q.4) Which of the following states has the highest % share of geographical area in the
Indian Himalayan Region (IHR)
a)
b)
c)
d)

Uttarakhand
Jammu & Kashmir
Himachal Pradesh
Arunachal Pradesh

Q.4) Solution (b)


Indian Himalayan States - http://gbpihedenvis.nic.in/him_states.htm

Q.5) Consider the following statements about Defence Communication Network (DCN)
1.
2.
3.
4.

It is the first tri-service communication and IT network of the Armed Forces


It is capable of working on terrestrial as well as satellite mode of communication
It has pan-India reach
It is designed and developed by HCL Infosystems

Select the correct statements


a)
b)
c)
d)

1 and 2 and 4
1, 2 and 3
2, 3 and 4
1, 2, 3 and 4

IASbabas Daily Quiz


November 28, 2016
Q.5) Solution (d)
The network enables the Army, the Air Force, the Navy and the Special Forces Command to
share situational awareness for a faster decision-making process. It is the first tri-service
communication and IT network of the Armed Forces.
It has a pan-India reach, i.e., from Ladakh to the North East to island territories.
It is capable of working on terrestrial as well as satellite mode of communication.
The network has been fully designed and developed in India by HCL Infosystems.

IASbabas Daily Quiz


November 29, 2016
Q.1) Consider the following statements about World Organisation for Animal Health (OIE)
1. The OIE is the intergovernmental organisation responsible for improving animal
health worldwide
2. It is recognised as a reference organisation by the World Trade Organization (WTO)
3. The World Assembly of Delegates is the highest authority of the OIE
Which of the following statements is/are correct?
a)
b)
c)
d)

Only 1
1 and 2
1 and 3
All of the above

Q.1) Solution (d)


The need to fight animal diseases at global level led to the creation of the Office
International des Epizooties through the international Agreement signed on January 25th
1924. In May 2003 the Office became the World Organisation for Animal Health but kept its
historical acronym OIE.
The OIE is the intergovernmental organisation responsible for improving animal health
worldwide.
It is recognised as a reference organisation by the World Trade Organization (WTO) and in
2016 has a total of 180 Member Countries. The OIE maintains permanent relations with 71
other international and regional organisations and has Regional and sub-regional Offices on
every continent.
The day-to-day operation of the OIE is managed at the Headquarters situated in Paris and
placed under the responsibility of a Director General elected by the World Assembly of
Delegates. The Headquarters implements the resolutions passed by the International
Committee and developed with the support of Commissions elected by the Delegates.
The World Assembly of Delegates is the highest authority of the OIE.
It comprises the Delegates of all Member Countries and meets at least once a year. The
General Session of the Assembly lasts five days and is held every year in May in Paris.
Voting by Delegates within the Assembly respects the democratic principle of 'one country,
one vote'.

IASbabas Daily Quiz


November 29, 2016
The OIE's financial resources are derived principally from compulsory annual contributions
backed up by voluntary contributions from Member Countries
In News - http://www.thehindu.com/sci-tech/health/H5N8-expands-itsreach/article16707475.ece

Q.2) Consider the following pairs


1.
2.
3.
4.

Free Basics :: Facebook


Internet Sathi :: Facebook
Express WiFi :: Google
Hyperloop :: Microsoft

Which of the pairs is/are incorrectly matched?


a)
b)
c)
d)

1, 2 and 4
2, 3 and 4
2 and 3
1 and 3

Q.2) Solution (b)


Googles Internet Saathi programme, under which Google agents go from village to village
and train women to use smart phones and the internet.
Social networking giant Facebook is now testing a new model for public Wi-Fi deployments
for offering quality internet access in rural parts of the country. It is called as Express Wifi.
Earlier Facebook has launched Free Basics in 2014 in partnership with Reliance
Communications for providing basic internet access to people. However Telecom Regulatory
authority has slammed the service saying it violated the principle of net neutrality.
Hyperloop is a new mode of passenger and freight transportation that propels a pod-like
vehicle through a near-vacuum tube at airline speeds. Developed by SpaceX.
In News - http://indianexpress.com/article/technology/tech-news-technology/facebookexpress-wi-fi-is-being-tested-in-india-for-over-a-year-heres-what-we-know-4399525/

IASbabas Daily Quiz


November 29, 2016
Q.3) Which of the following places is the largest market for tomatoes in Asia?
a)
b)
c)
d)

Pimpalgaon, Nashik, Maharashtra


Kolar, Karnataka
Khargpur, West Bengal
Forberganj, Bihar

Q.3) Solution (a)


In News - http://www.thehindubusinessline.com/economy/agri-business/kolar-tomatomarket/article9387022.ece

Q.4) Consider the following statements regarding Shale gas exploration in India.
1. Shale gas is natural gas that is trapped within shale formations, which are essentially
fine-grained sedimentary rocks
2. Andhra Pradesh leads in the shale gas exploration
3. Fracking leads to contamination of drinking water and soil with toxic and
carcinogenic chemicals and also causes mild tremors in the region
Which of the following statements is/are correct?
a)
b)
c)
d)

1 and 2
2 and 3
1 and 3
All of the above

Q.4) Solution (c)


Gujarat (28 blocks) leads in the shale gas exploration followed by Andhra Pradesh (10
blocks)
Shale refers to a sedimentary rock resulted from compaction process of small old rocks
containing mud and minerals such as quartz and calcite, trapped beneath the earth
surface. The shale gas is the natural gas trapped within these shale rocks. It is one of the
unconventional type of natural gas along with coal bed methane, tight sandstones, and
methane hydrates.
Scientists and environmentalists warn of the potential consequences on the environment.

IASbabas Daily Quiz


November 29, 2016
The process involves huge amounts of water, and as argued by Chandan Nandy in TOI, India
cannot afford to waste such large quantities of water, that can otherwise be used for
drinking or farming.
Besides causing mild tremors in the region, fracking has in the past led to contamination of
drinking water and soil with toxic and carcinogenic chemicals.
Fracking has been banned in France, Netherlands, Ireland, and Scotland and in some states
in the United States.
In News - http://timesofindia.indiatimes.com/city/hyderabad/Extraction-of-shale-gasfrom-KG-basin-may-trigger-tremors/articleshow/55630449.cms?null

Q.5) Consider the following statements about National Culture Fund (NCF)
1. It enables the government to mobilize extra budgetary resources by allowing direct
contributions to the NCF account maintained outside the Consolidated Fund of India
2. The NCF is managed and administered by a council chaired by the Minister for
Tourism & Culture
3. Donations to NCF will be eligible 100 per cent tax rebate under Section 80G of the
Income Tax Act
Select the correct statements
a)
b)
c)
d)

Only 2
1 and 2
2 and 3
All of the above

Q.5) Solution (d)


The National Culture Fund (NCF) was established as a funding mechanism distinct from the
existing sources and patterns of funding for the arts and culture in India. It will enable
institutions and individuals to support arts and culture directly as partners with its
government.
The National Culture Fund (NCF) was created as a Trust in November 1996.
The Govt. of India vide their orders in Aug.-Sep.98 notified that the donations to the
national Culture Fund will be eligible for tax benefit under section 10 (23C) (iv) and 80 G(2)
of the Income Tax Act.

IASbabas Daily Quiz


November 29, 2016
The NCF is managed and administered by a council to decide the policies and an Executive
Committee to actualize those policies. The Council is chaired by the Union Minister of
Tourism & Culture and has a maximum strength of 24 including both the Chairman and
Member Secretary, A team of 19 members represent various fields including corporate
sector, private foundations and not-for-profit voluntary organizations. The purpose for this
structure is to increase non-government representation in the decision making process.

IASbabas Daily Quiz


November 30, 2016
Q.1) Consider the following statements about Insolvency and Bankruptcy Board of India
(IBBI)
1. It regulates the functioning of insolvency professionals, insolvency professional
agencies and information utilities under the Insolvency and Bankruptcy Code 2016
2. The Board members include people from RBI, Ministry of Finance, Ministry of Law
and Ministry of corporate Affairs
Which of the following statements is/are incorrect?
a)
b)
c)
d)

Only 1
Only 2
Both 1 and 2
Neither 1 nor 2

Q.1) Solution (d)


The Insolvency and Bankruptcy Board of India was established on October 1, 2016 in
accordance with the provisions of The Insolvency and Bankruptcy Code, 2016.
he Preamble of the Insolvency and Bankruptcy Code describes the basic functions of the
Insolvency and Bankruptcy Code as "...to consolidate and amend the laws relating to
reorganization and insolvency resolution of corporate persons, partnership firms and
individuals in a time bound manner for maximization of the value of assets of such persons,
to promote entrepreneurship, availability of credit and balance the interests of all the
stakeholders including alteration in the order of priority of payment of Government dues
and to establish an Insolvency and Bankruptcy Code of India, and for matters connected
therewith or incidental thereto."
The Board members include Secretaries from Ministry of Law, Finance and Corporate Affairs
and Legal Adviser from RBI.
In News - http://economictimes.indiatimes.com/news/economy/policy/insolvency-andbankruptcy-board-sets-up-two-advisory-panels/articleshow/55649609.cms

Q.2) Consider the following statements about United Nations Commission on


International Trade Law (UNCITRAL)
1. It was established to promote the progressive unification and harmonisation of
international trade law
2. It is the core legal body of the UN system in the field of international trade law

IASbabas Daily Quiz


November 30, 2016
3. India has been a member of UNCITRAL since its inception
Select the correct statements
a)
b)
c)
d)

Only 1
1 and 2
1 and 3
1, 2 and 3

Q.2) Solution (d)


It is the core legal body of the UN system in the field of international trade law. It specialises
in reforms in commercial law worldwide for a period of over 40 years. India is a founding
member of UNCITRAL. India is only one of eight countries which have been a member of
UNCITRAL since its inception.
In News - http://indianexpress.com/article/business/business-others/improved-legalstandards-needed-to-deal-with-trade-disputes-says-mea-4401963/

Q.3) Recently the Chinese diplomats have been using the famous traveller Hiuen Tsangs
legacy to rekindle cooperation between India and China. Consider the following
statements with regard to Hiuen Tsang
1. He was known as Prince of Pilgrims
2. He travelled India during the reign of Chandra Gupta II
Choose the appropriate code
a)
b)
c)
d)

Only 1
Only 2
Both 1 and 2
Neither 1 nor 2

Q.3) Solution (a)


Hiuen Tsang (also Xuanzang, Hsuan Tsang) was the celebrated Chinese traveler who visited
India in Ancient Times. He has been described therefore as the Prince of Pilgrims.

IASbabas Daily Quiz


November 30, 2016
His visit to India was an important event of the reign of Harshavardhana. India is much
indebted to this Chinese traveller for the valuable accounts he left behind with many details
of political, religious, economic, social conditions of those days.
Harsha (c. 590648), also known as Harshavardhana, was an Indian emperor who ruled
North India from 606 to 648 from his capital Kannauj. He was the founder and ruler of the
Empire of Harsha. Harshavardhana was the last Hindu emperor of united North India.
Harsha is widely believed to be the author of three Sanskrit plays Ratnavali, Nagananda and
Priyadarsik
In News - http://www.business-standard.com/article/pti-stories/china-keen-to-discoversites-mentioned-by-hiuen-tsang-116110401356_1.html

Q.4) In Indian context, protection of the cultural heritage of India is a part of


1) Directive Principles of State Policy
2) Fundamental Duties
3) Archaeological Survey of India
Choose the appropriate code
a)
b)
c)
d)

2 only
1 only
2 and 3
1, 2 and 3

Q.4) Solution (d)


51 (f) - To value and preserve the rich heritage of our composite culture;
Article 49 places an obligation upon the State to ensure the preservation of monuments and
objects of national importance
The Archaeological Survey of India (ASI), under the Ministry of Culture, is the premier
organization for the archaeological researches and protection of the cultural heritage of the
nation. Maintenance of ancient monuments and archaeological sites and remains of
national importance is the prime concern of the ASI.

IASbabas Daily Quiz


November 30, 2016
Q.5) Consider the following statements with respect to coal
1. The value of coal depends on the concentration of carbon in its composition
2. Bituminous Coal has the highest calorific value
3. Anthracite Coal is not found in India
Select the correct statements
a)
b)
c)
d)

Only 1
1 and 2
2 and 3
1 and 3

Q.5) Solution (a)


Anthracite coal has the highest calorific value and it is found in India.

You might also like